- PowerScore Staff
- Posts: 5972
- Joined: Mar 25, 2011
- Sun Jan 21, 2018 12:00 am
#78446
Complete Question Explanation
(The complete setup for this game can be found here: lsat/viewtopic.php?t=15592)
The correct answer choice is (B)
The question stem indicates that the computer in office 3 was bought in an earlier year than the printer in office 3. Because the printer in office 3 was purchased in 1988, we can infer that the computer in office 3 was purchased in 1987. In addition, from the third rule we can then infer that the printer in office 4 was purchased in 1987. Finally, from the first rule, we can infer that the computer in office 4 was purchased in 1987:
Accordingly, answer choice (B) could be true and is correct.
(The complete setup for this game can be found here: lsat/viewtopic.php?t=15592)
The correct answer choice is (B)
The question stem indicates that the computer in office 3 was bought in an earlier year than the printer in office 3. Because the printer in office 3 was purchased in 1988, we can infer that the computer in office 3 was purchased in 1987. In addition, from the third rule we can then infer that the printer in office 4 was purchased in 1987. Finally, from the first rule, we can infer that the computer in office 4 was purchased in 1987:
Accordingly, answer choice (B) could be true and is correct.
You do not have the required permissions to view the files attached to this post.
Dave Killoran
PowerScore Test Preparation
Follow me on X/Twitter at http://twitter.com/DaveKilloran
My LSAT Articles: http://blog.powerscore.com/lsat/author/dave-killoran
PowerScore Podcast: http://www.powerscore.com/lsat/podcast/
PowerScore Test Preparation
Follow me on X/Twitter at http://twitter.com/DaveKilloran
My LSAT Articles: http://blog.powerscore.com/lsat/author/dave-killoran
PowerScore Podcast: http://www.powerscore.com/lsat/podcast/